Difference between revisions of "2021 Fall AMC 12A Problems/Problem 7"

(As I found out what t and s are, I just want to make the solution more concise by not substituting at the very end. It should be clear to readers already.)
(The new edits are not in original wording.)
 
(12 intermediate revisions by 3 users not shown)
Line 21: Line 21:
  
 
~MRENTHUSIASM
 
~MRENTHUSIASM
 +
 +
==Video Solution (Simple and Quick)==
 +
https://youtu.be/nZBIoUS0mdk
 +
 +
~Education, the Study of Everything
 +
  
 
==Video Solution by TheBeautyofMath==
 
==Video Solution by TheBeautyofMath==
Line 30: Line 36:
  
 
==Video Solution by WhyMath==
 
==Video Solution by WhyMath==
https://youtu.be/f7vhOCnvl0k ~savannahsolver
+
https://youtu.be/f7vhOCnvl0k  
 +
 
 +
~savannahsolver
  
 
==See Also==
 
==See Also==

Latest revision as of 03:02, 13 July 2024

The following problem is from both the 2021 Fall AMC 10A #10 and 2021 Fall AMC 12A #7, so both problems redirect to this page.

Problem

A school has $100$ students and $5$ teachers. In the first period, each student is taking one class, and each teacher is teaching one class. The enrollments in the classes are $50, 20, 20, 5,$ and $5$. Let $t$ be the average value obtained if a teacher is picked at random and the number of students in their class is noted. Let $s$ be the average value obtained if a student was picked at random and the number of students in their class, including the student, is noted. What is $t-s$?

$\textbf{(A)}\ {-}18.5  \qquad\textbf{(B)}\ {-}13.5 \qquad\textbf{(C)}\ 0 \qquad\textbf{(D)}\ 13.5 \qquad\textbf{(E)}\ 18.5$

Solution

The formula for expected values is \[\text{Expected Value}=\sum(\text{Outcome}\cdot\text{Probability}).\] We have \begin{align*} t &= 50\cdot\frac15 + 20\cdot\frac15 + 20\cdot\frac15 + 5\cdot\frac15 + 5\cdot\frac15 \\ &= (50+20+20+5+5)\cdot\frac15 \\ &= 100\cdot\frac15 \\ &= 20, \\ s &= 50\cdot\frac{50}{100} + 20\cdot\frac{20}{100} + 20\cdot\frac{20}{100} + 5\cdot\frac{5}{100} + 5\cdot\frac{5}{100} \\ &= 25 + 4 + 4 + 0.25 + 0.25 \\ &= 33.5. \end{align*} Therefore, the answer is $t-s=\boxed{\textbf{(B)}\ {-}13.5}.$

~MRENTHUSIASM

Video Solution (Simple and Quick)

https://youtu.be/nZBIoUS0mdk

~Education, the Study of Everything


Video Solution by TheBeautyofMath

for AMC 10: https://youtu.be/ycRZHCOKTVk?t=789

for AMC 12: https://youtu.be/wlDlByKI7A8?t=157

~IceMatrix

Video Solution by WhyMath

https://youtu.be/f7vhOCnvl0k

~savannahsolver

See Also

2021 Fall AMC 12A (ProblemsAnswer KeyResources)
Preceded by
Problem 6
Followed by
Problem 8
1 2 3 4 5 6 7 8 9 10 11 12 13 14 15 16 17 18 19 20 21 22 23 24 25
All AMC 12 Problems and Solutions
2021 Fall AMC 10A (ProblemsAnswer KeyResources)
Preceded by
Problem 9
Followed by
Problem 11
1 2 3 4 5 6 7 8 9 10 11 12 13 14 15 16 17 18 19 20 21 22 23 24 25
All AMC 10 Problems and Solutions

The problems on this page are copyrighted by the Mathematical Association of America's American Mathematics Competitions. AMC logo.png